5
$\begingroup$

Let $X$ be a Gaussian random variable with mean=variance=$g>0$, namely its probability density is $p(x)=\frac{1}{\sqrt{2\pi g}}\,e^{-\frac{(x-g)^2}{2g}}$. I am looking for proofs of the following fact: $$ I(g):=\mathbb E[(1-3\tanh^2 X)\,(1-\tanh^2 X)^2] \,\geq\,0 .$$

I have one proof but I am looking also for other ones. My proof relies on the fact that $$ i:=\int_0^1 (1-3y^2) (1-y^2)^\frac{1}{2} dy \,>\, 0$$ since it can be computed explicitly. Now changing variable $\tanh(x)=y$ and expanding the square in the gaussian density you can rewrite $$ I(g) = \frac{e^{-\frac{g}{2}}}{\sqrt{2\pi g}}\, \int_{-1}^1 (1-3y^2)\,(1-y^2)^2 \,e^{\operatorname{atanh}(y)}\,e^{-\frac{1}{2g}\operatorname{atanh}^2(y)} \frac{d y}{1-y^2}$$ then, using $e^{\operatorname{atanh}(y)}+e^{\operatorname{atanh}(-y)}=\frac{2}{\sqrt{1-y^2}}$, you get $$ I(g) = \frac{2\,e^{-\frac{g}{2}}}{\sqrt{2\pi g}}\, \int_{0}^1 (1-3y^2)\,(1-y^2)^\frac{1}{2} \,e^{-\frac{1}{2g}\operatorname{atanh}^2(y)} \,d y $$ Finally, using the fact that $e^{-\frac{1}{2g}\operatorname{atanh}(y)^2}$ is monotonically decreaing in $y\in[0,1]$, you can bound with the value at the point $y=\frac{1}{\sqrt{3}}$ where the integrand changes sign: $$ I(g) \geq \frac{2\,e^{-\frac{g}{2}}}{\sqrt{2\pi g}}\, e^{-\frac{1}{2g}\operatorname{atanh}^2(\frac{1}{\sqrt{3}})} \ \int_{0}^1 (1-3y^2)\,(1-y^2)^\frac{1}{2} \,d y$$ and you conclude since you know $i>0$.

$\endgroup$
1
  • 3
    $\begingroup$ Great job. Also, I'm not the one who downvoted. Let me upvote instead :) $\endgroup$ Commented 17 hours ago

2 Answers 2

8
$\begingroup$

We have

$$\begin{align*} I(g) &= \mathbb{E}\biggl[ \frac{1-2\sinh^2 X}{(1+\sinh^2 X)^3} \biggr] \\ &= \frac{1}{\sqrt{2\pi g}} \int_{-\infty}^{\infty} \frac{1-2\sinh^2 x}{(1+\sinh^2 x)^3} e^{-(x-g)^2/2g} \, \mathrm{d}x \\ &= \frac{e^{-g/2}}{\sqrt{2\pi g}} \int_{-\infty}^{\infty} \frac{1-2\sinh^2 x}{(1+\sinh^2 x)^3} e^{-x^2/2g} \cosh x \, \mathrm{d}x. \end{align*}$$

The last line follows by writing $e^x = \cosh x + \sinh x$ and noting that the term involving $\sinh x$ is an odd function. Now, here comes the crucial observation:

$$ \int \frac{1-2\sinh^2 x}{(1+\sinh^2 x)^3} \cosh x \, \mathrm{d}x = F(x) + \mathsf{C}, $$

where $F(x)$ is defined by

$$ F(x) = \frac{1}{8} \arctan(\sinh x) + \frac{1}{8} \frac{\sinh x}{1+\sinh^2 x} + \frac{3}{4}\frac{\sinh x}{(1+\sinh^2 x)^2}. $$

This is easily proved by substituting $u = \sinh x$. So by performing integration by parts,

$$\begin{align*} I(g) &= \frac{e^{-g/2}}{\sqrt{2\pi} g^{3/2}} \int_{-\infty}^{\infty} xF(x) e^{-x^2/2g} \, \mathrm{d}x. \end{align*}$$

Since the integrand is non-negative, the desired claim follows.

$\endgroup$
1
  • $\begingroup$ Very nice, you just used integration by parts in the end $\endgroup$ Commented 15 hours ago
1
$\begingroup$

By $$\int \left(1-3 \tanh ^2(x)\right) \left(1-\tanh ^2(x)\right) \, dx=\frac{\sinh (x)}{ \cosh^3(x)}$$ and ignoring boundary values with the Gaussian measure at $\pm \infty$ integration by parts yields a positive integrand $$ e^{-\frac{x^2}{2 \sigma ^2}} \ x \ \frac{\sinh (x)}{ \cosh^3(x)}\ \ \, dx $$

$\endgroup$
2
  • $\begingroup$ Aren't you forgetting the $e^x$ term in the density? $\endgroup$ Commented 14 hours ago
  • $\begingroup$ Transfer the shift of $(x-a) e^{-(x-a)^2}$ to the to the arguments of the hyperbolic expression. $\endgroup$ Commented 9 hours ago

You must log in to answer this question.

Start asking to get answers

Find the answer to your question by asking.

Ask question

Explore related questions

See similar questions with these tags.